27.09.2017 Views

Internal-Medicine

Create successful ePaper yourself

Turn your PDF publications into a flip-book with our unique Google optimized e-Paper software.

CHAPTER 1<br />

Cardiology<br />

Questions<br />

DIRECTIONS (Questions 1 through 61): Each of<br />

the numbered items in this section is followed by<br />

answers. Select the ONE lettered answer that is<br />

BEST in each case.<br />

1. A 62-year-old man with coronary artery disease<br />

(CAD) presents with presyncope. His<br />

physical examination is normal except for<br />

bradycardia (pulse 56 beats/min) and an irregular<br />

pulse. The electrocardiogram (ECG) shows<br />

Wenckebach’s type atrioventricular (AV) block.<br />

Which of the following are you most likely to<br />

see on the ECG?<br />

(A)<br />

(B)<br />

(C)<br />

(D)<br />

(E)<br />

progressive PR shortening<br />

progressive lengthening of the PR interval<br />

tachycardia<br />

dropped beat after PR lengthening<br />

fixed 2:1 block<br />

2. A 72-year-old woman had a pacemaker<br />

inserted 4 years ago for symptomatic bradycardia<br />

because of AV nodal disease. She is clinically<br />

feeling well and her ECG shows normal<br />

sinus rhythm at a rate of 68/min but no pacemaker<br />

spikes. Her pacemaker only functions<br />

when the ventricular rate falls below a preset<br />

interval. Which of the following best describes<br />

her pacemaker function?<br />

3. A 42-year-old man develops shortness of breath<br />

(SOB) and chest pain 7 days after an open<br />

cholecystectomy. His blood pressure is 145/86<br />

mm Hg, pulse is 120/min, respirations 24/min,<br />

and oxygen saturation of 97%. Pulmonary<br />

embolism is clinically suspected. Which of the<br />

following is the most common ECG finding of<br />

pulmonary embolism?<br />

(A) a deep S wave in lead I<br />

(B) depressed ST segments in leads I and II<br />

(C) prominent Q wave in lead I, and<br />

inversion of T wave in lead III<br />

(D) sinus tachycardia<br />

(E) clockwise rotation in the precordial<br />

leads<br />

4. A 63-year-old woman develops exertional<br />

angina and has had two episodes of syncope.<br />

Examination shows a systolic ejection murmur<br />

with radiation to the carotids and a soft S 2<br />

. Which<br />

of the following is the most likely diagnosis?<br />

(A)<br />

(B)<br />

(C)<br />

(D)<br />

(E)<br />

mitral stenosis<br />

mitral insufficiency<br />

aortic stenosis<br />

aortic insufficiency<br />

tricuspid stenosis<br />

(A)<br />

(B)<br />

(C)<br />

(D)<br />

(E)<br />

asynchronous<br />

atrial synchronous<br />

ventricular synchronous<br />

ventricular inhibited<br />

atrial sequential<br />

1<br />

Copyright © 2007 by The McGraw-Hill Companies, Inc. Click here for terms of use.

Hooray! Your file is uploaded and ready to be published.

Saved successfully!

Ooh no, something went wrong!